Sie sind auf Seite 1von 8

PHY4604 Final Exam Solutions

PHY 4604 Final Exam Solutions


Monday December 5, 2005 (Total Points = 100)

Problem 1 (10 points): Circle true or false for following (1 point each).

(a) (True or False) If the expectation value of the operator Oop is a real number then Oop is
hermitian.

(b) (True or False) If |ψ> is a (normalized) “ket” vector, then the operator Oop = |ψ><ψ| has the
property that (Oop)2 = 1.

(c) (True or False) If the operator Uop is unitary then, its inverse is equal it its hermitian
conjugate.

(d) (True or False) If Pop is the parity operator and if Oop± = 12 (1 ± Pop ) , then (Oop± ) 2 = Oop± .

(e) (True or False) One can simultaneously know the precise values of commuting observables.

(f) (True or False) The overall wave function (space × spin) for two identical spin 3/2 particles is
always symmetric under the interchange of the two particles.

(g) (True or False) A µ- particle has the same charge as an electron but its mass is 207 times
greater, hence the ground state energy of a muonic atom (µ- and a proton) is lower than the
ground state of hydrogen.

(h) (True or False) In SU(2), 3 × 3 = 4 + 3 + 2.

(i) (True or False) If one neglects the electron-electron interactions then the ground state of
helium would have a degeneracy of 2.

(j) (True or False) The maximum number of electrons that can be in the nth energy level of an
atom is n2.

Department of Physics Page 1 of 8


PHY4604 Final Exam Solutions
y
Problem 2 (30 points): Consider a bead of mass m that slides frictionlessly
around a circular wire ring of radius R. The bead is constrained to lie on the R
s
circle. This is similar to the free particle with Hamiltonian
θ
p2 h2 d 2 h2 d 2
H= =− =− . x
2m 2m ds 2 2mR 2 dθ 2
where s = Rθ is the distance around the circle. We look for stationary state
solutions of the form Ψ (θ , t ) = ψ (θ )e − iEt / h with the time-independent
Schrödinger equation given by
h 2 d 2ψ
− = Eψ .
2mR 2 dθ 2
Consider separately the solutions corresponding to clockwise, ψ + , and counter-clockwise, ψ − ,
movement around the wire. The probability density ρ ± (θ ) =| ψ ± |2 is the probability of finding
the particle between θ and θ+dθ and is normalized as follows:

∫ρ (θ )dθ = 1 .
±

(a) (20 points) Find the stationary states ψ ± (with the correct normalization) and the
corresponding allowed energies. (Hint: requireψ (θ + 2π ) = ψ (θ ) since s and s+2πR correspond
to the same point on the wire)
1 m inθ ± h 2 n 2
Answer: ψ n (θ ) =
±
e , En = (n = 0, 1, 2, …)
2π 2mR 2
Solution: We see that
d 2ψ 2mR 2 E h 2α 2
= − α 2
ψ with α 2
= and E =
dθ 2 h2 2mR 2
with solution
ψ (θ ) = Ae + iαt + Be − iαt
For α > 0 the first term corresponds to counter-clockwise movement and the second term
corresponds to clockwise movement. We also know that
ψ (θ + 2π ) = Ae + iα (θ + 2π ) + Be −iα (θ + 2π ) = ψ (θ ) = Ae + iαθ + Be −iαθ
which implies that
Ae + iαθ (e + iα 2π − 1) + Be − iαθ (e − iα 2π − 1) = 0 .
This equation implies that e ± iα 2π = 1 which implies that α = n = 0, ±1, ±2, …and hence
1 − inθ h 2n2
ψ n (θ ) =
+
e and En = +
(n = 0, 1, 2, …)
2π 2mR 2
1 + inθ h 2n2
ψ n− (θ ) = e and En− = (n = 0, 1, 2, …).
2π 2mR 2
The normalization comes from
2π 2π

∫ ρ (θ )dθ = 1 = ∫ | ψ | dθ = 2πA2
+ 2
n n
0 0

Note that α = kR = n, which implies that 2πR = nλ, where n = 0, ±1, ±2, ....

Department of Physics Page 2 of 8


PHY4604 Final Exam Solutions

(b) (5 points) What is the lowest allowed energy and how many states have this energy and what
are there wave functions?
1
Answer: ψ 0± (θ ) = , E0± = 0 (n = 0)

(c) (5 points) Are the stationary states also eigenfunctions of the z-component of the angular
d
momentum operator Lz = −ih ? Is so what are the eigenvalues of Lz?

Answer: yes, Lzψ n± = m nhψ n±
Solution: We see that
d
Lzψ n± = −ih ψ n± (θ ) = m nhψ n± .

r
Problem 3 (30 points): The Hamiltonian of a charged particle with intrinsic spin S at rest in a
r r r r q
magnetic field is H = − µ ⋅ B = −γS ⋅ B , where γ = (q is the electric charge, m is the mass).
m
r
For spin 1 we have S = S x xˆ + S y yˆ + S z zˆ with
0 0 0   0 0 1  0 −1 0
     
S x = ih 0 0 − 1 S y = ih 0 0 0  S z = ih  1 0 0 
0 1 0   −1 0 0  0 0 0
     
(a) (10 points) Consider the vector boson, W+, with charge q = +e and mass MW. Suppose a W+
r
is at rest in a uniform magnetic field which points in the z-direction, B = B0 zˆ . What are the
energy levels and the corresponding eigenkets of the system? What is the ground state energy
and ground state eigenket?
Answer: The ground state (i.e. lowest) energy is E0 = −hγB0 with eigenket
1   0
1   st  
| χ 0 >=| 1 − 1 >=  − i  . The 1 excited state has energy E1 = 0 with eigenket | χ1 >=| 10 >=  0  .
2  1 
0   
1 
1  
The 2 excited state has energy E2 = +hγB0 with eigenket | χ 2 >=| 11 >=
nd
i  .
2 
 0
Solution: The eigenvalues of Sz are determined from
− λ − ih 0
ih − λ 0 = 0 and hence − λ (λ ) + h λ = λ (h − λ ) = 0
2 2 2 2

0 0 −λ
Thus there are three eigenvalues λ = h,0,−h . The eigenket corresponding to λ = 0 is determined
from
 0 − 1 0  a   − b  0  0
        
ih 1 0 0  b  = ih a  =  0  which implies that a = b = 0 and | 10 >=  0 
 0 0 0  c   0   0 1
        

Department of Physics Page 3 of 8


PHY4604 Final Exam Solutions

The eigenkets corresponding to λ = ±h are determined from


 0 − 1 0  a  − b a
      
ih 1 0 0  b  = ih a  = ±h b  which implies that ±a = -ib and hence
 0 0 0  c   0  c
      
1  1 
1   1  
| 11 >=  i  and | 1 − 1 >=  − i  and where S z | 1m >= mh | 1m > .
2  2 
0 0 
r r
The Hamiltonian is given by H = −γS ⋅ B = −γB0 S z . Hence there are three energy levels
E0 = −hγB0 , E1 = 0 , E2 = +hγB0 . The ground state (i.e. lowest) energy is E0 = −hγB0 with
eigenket
1 
1  
| χ 0 >=| 11 >= i  .
2 
 0
The 1st excited state has energy E1 = 0 with eigenket
 0
 
| χ1 >=| 10 >=  0  .
1 
 
The 2nd excited state has energy E2 = +hγB0 with eigenket
1 
1  
| χ 2 >=| 1 − 1 >= −i.
2 
0 
(b) (10 points) Suppose that at t = 0 the W+ boson is in the state
1
1  
| χ (0) >=  0 .
2 
i
What is | χ (t ) > ? If I measure the energy of the state | χ (t ) > , what values might I get and what
is the probability of getting these values.
 cos(γB0t ) 
1  
Answer: | χ (t ) >=  − sin(γB0t )  , we will measure E0 = −hγB0 with probability P0 = 4 and
1

2 
 i 
we will measure E1 = 0 with probability P1 = 12 and we will measure E2 = +hγB0 with probability
P2 = 14 .
Solution: We know that
| χ (t ) >= a0 | χ 0 > e − iE0 t / h + a1 | χ1 > e −iE1t / h + a2 | χ 2 > e − iE 2 t / h
where

Department of Physics Page 4 of 8


PHY4604 Final Exam Solutions

1
1 1   1
a0 =< χ 0 | χ (0) >= (1 − i 0) 0 =
2 2  2
i
1
1   i
a1 =< χ1 | χ (0) >= (0 0 1) 0 =
2  2
i
1
1 1   1
a2 =< χ 2 | χ (0) >= (1 i 0) 0 =
2 2  2
i
Hence we will measure E0 with probability P0 =| a0 |2 = 1
4
and we will measure E1 with
probability P1 =| a1 | = and we will measure E2 with probability P2 =| a2 |2 = 14 . Also,
2 1
2

| χ (t ) >= a0 | χ 0 > e + iγB0 t + a1 | χ1 > + a2 | χ 2 > e − iγB0 t


1  0 1
1   i  
+ iγB0 t 1 − iγB0 t  
= e i+  0 + e − i
2 2 0 2  2 2 0
  1  
 12 (e 0 + e 0 ) 
+ i γB t − i γ B t
cos(γB0t ) 
1  1 + i γB 0 t − iγB0 t
 1  
=  2 (ie − ie ) =  − sin(γB0t ) 
2  2 
 i   i 
(c) (10 points) What is <Sx(t)>, <Sy(t)>, and <Sz(t)> in the state | χ (t ) > ? Express your answer
in terms of ω = γB0 .
Answer: < S x (t ) >= −h sin(ωt ) , < S y (t ) >= −h cos(ωt ) , < S z (t ) >= 0
Solution: We see that

< S x (t ) >=< χ (t ) | S x | χ (t ) >=


 0 0 0  cos(ωt ) 
1   
(cos(ωt ) − sin(ωt ) − i )ih 0 0 − 1 − sin(ωt ) 
2  0 1 0  
  i 
 0 
ih  
= (cos(ωt ) − sin(ωt ) − i ) − i  = −h sin(ωt )
2  − sin(ωt ) 
 

Department of Physics Page 5 of 8


PHY4604 Final Exam Solutions

< S y (t ) >=< χ (t ) | S y | χ (t ) >=


 0 0 1  cos(ωt ) 
1   
(cos(ωt ) − sin(ωt ) − i )ih 0 0 0  − sin(ωt ) 
2  − 1 0 0  
  i 
 i 
ih  
= (cos(ωt ) − sin(ωt ) − i ) 0  = −h cos(ωt )
2  − cos(ωt ) 
 
< S z (t ) >=< χ (t ) | S z | χ (t ) >=
 0 − 1 0  cos(ωt ) 
1   
(cos(ωt ) − sin(ωt ) − i )ih 1 0 0  − sin(ωt ) 
2  0 0 0  
  i 
 sin(ωt ) 
ih  
= (cos(ωt ) − sin(ωt ) − i ) cos(ωt )  = 0
2  0 
 

Problem 4 (30 points): Consider the case of two non-interacting particles both with mass m in a
one-dimensional infinite square well given by V(x) = 0 for 0 < x < L, and V(x) = ∞ elsewhere.
(a) (15 points) If the two particles are identical spin 0 bosons, what are the energy levels of the
system? What is the energy, wave functions (correctly normalized) and degeneracy (i.e. number
of states) for the ground state and the 1st excited state?
Solution: For one particle the stationary states of Schrödinger’s equation are given by
2
Ψn ( x, t ) = ψ n ( x)e −iE n t / h with ψ n ( x) = sin(nπx / L)
L
π 2h 2
and En = n 2 E0 , and n is a positive integer and E0 = . For two identical bosons the overall
2mL2
wave function must be symmetric so for α ≠ β we have
ψ αβ
S
( x1 , x2 ) =
1
(ψ αβ ( x1 , x2 ) + ψ αβ ( x2 , x1 ) )
2
2
= [sin(απx1 / L) sin( βπ x2 / L) + sin( βπx1 / L) sin(απx2 / L)]
L
and for α = β we have
1
ψ αα
S
( x1 , x2 ) = (ψ αα ( x1 , x2 ) + ψ αα ( x2 , x1 ) ) = ψ αα ( x1 , x2 )
2
2
= sin(απx1 / L ) sin(απx2 / L )
L
The energy levels are
Eαβ = ( E1 )α + ( E2 ) β = (α 2 + β 2 ) E0
with α = 1, 2, 3, … and β = 1, 2, 3, … . The ground state corresponds to α = β = 1 and has
energy E0Bosons = 2E0 and wave function

Department of Physics Page 6 of 8


PHY4604 Final Exam Solutions

2
ψ 11Bosons ( x1 , x2 ) =
sin(πx1 / L ) sin(πx2 / L ) (1 state)
L
There is only one ground state (i.e. degeneracy 1). The 1st excited state has α = 1 and β = 2 or α
= 2 and β = 1 and has energy E1Bosons = 5E0 and wave functions
2
ψ 12Bosons ( x1 , x2 ) = [sin(πx1 / L) sin( 2πx2 / L) + sin( 2πx1 / L ) sin(πx2 / L)] (1 state)
L
ψ 21Bosons ( x1 , x2 ) = ψ 12Bosons ( x1 , x2 ) (indistinguishable - same state)
The 1st excited state also has a degeneracy of 1.
(b) (15 points) If the two particles are identical spin 1/2 fermions, what are the energy levels of
the system? What is the energy, wave functions (both space and spin correctly normalized) and
degeneracy (i.e. number of states) for the ground state and the 1st excited state?
Solution: The possible spin states for two spin ½ fermions (i.e. s1 = ½, s2 = ½) are as follows:
χ11 =| 11 >=|↑↑>

χ10 =| 10 >=
1
(|↑↓> + |↓↑>) χ 00 =| 00 >=
1
(|↑↓> − |↓↑>)
2 2
χ1−1 =| 1 − 1 >=|↓↓>
r r r
The total spin s = s1 + s2 is either s = 1 (i.e. the “triplet” χ mT ) or s = 0 (i.e. the “singlet” χS). Note
that “triplet” states χ mT (m = 1, 0, -1) are symmetric under the exchange of the two electrons and
“singlet” state is anti-symmetric under the exchange of the two electrons. We know that the
overall wave function for two identical fermions must be anti-symmetric so we have the
following three possibilities. For α ≠ β we have
ψ αβ
Para
= ψ αβ
S
( x1 , x2 ) χ S =
1
(ψ αβ ( x1 , x2 ) + ψ αβ ( x2 , x1 ) )χ S
2
2
= [sin(απx1 / L) sin( βπ x2 / L) + sin( βπ x1 / L) sin(απx2 / L)]χ S
L
and if α = β we get
2
ψ αα
Para
= ψ αα
S
( x1 , x2 ) χ S = ψ αα ( x1 , x2 ) χ S = sin(απx1 / L ) sin(απx2 / L ) χ S
L
The other possibility is
ψ αβ m = ψ αβ ( x1 , x2 ) χ m =
Ortho A T 1
(ψ αβ ( x1 , x2 ) − ψ αβ ( x2 , x1 ) )χ mT
2
2
= [sin(απx1 / L) sin( βπ x2 / L) − sin( βπ x1 / L) sin(απx2 / L)]χ mT
L
The energy levels are
Eαβ = ( E1 )α + ( E2 ) β = (α 2 + β 2 ) E0
with α = 1, 2, 3, … and β = 1, 2, 3, … . The ground state is “para” and corresponds to α = β = 1
and has energy E0Fermions = 2E0 and wave function
2
ψ 11Para = ψ αβ
S
( x1 , x2 ) χ S = sin(πx1 / L ) sin(πx2 / L ) χ S (1 state)
L

Department of Physics Page 7 of 8


PHY4604 Final Exam Solutions

There is only one ground state (i.e. degeneracy 1). The 1st excited state is the “para” or “ortho”
state with α = 1 and β = 2 or α = 2 and β = 1 and has energy E1Fermions = 5E0 and wave functions
are
2
ψ 12Para = [sin(πx1 / L) sin( 2πx2 / L) + sin( 2πx1 / L ) sin(πx2 / L)]χ S (1 state)
L
ψ 21Para = −ψ 12Para (same state)
2
ψ 12Ortho
m = [sin(πx1 / L) sin( 2πx2 / L ) − sin( 2πx1 / L) sin(πx2 / L )]χ mT (3 states, m = 1, 0, -1)
L
ψ 21Ortho
m = −ψ 12 m
Ortho
(same 3 states)
st
The 1 excited state has a degeneracy of 4 (i.e. one “para” state and three “ortho” states)..

Department of Physics Page 8 of 8

Das könnte Ihnen auch gefallen